terça-feira, 10 de julho de 2012

Portfolio I Capítulo 22 - Física Geral II


Capítulo 22 - Campo elétrico

1. Na figura, as linhas do campo elétrico do lado esquerdo têm uma separação duas vezes maior que as linhas do lado direito.
a) Se o módulo do campo elétrico no ponto A é 40N/C, qual é o módulo da força a que é submetido um próton no ponto A?
b) Qual é o módulo do campo elétrico no ponto B?
Esse exercício deve ser o único realmente fácil do capítulo 22, junto com o 5. Inclusive, esse é realmente fácil. Só precisamos saber de duas informações para resolver a A:
$$F = Eq \\
q_{p} = 1.6*10^{-19}C$$
No caso, que a força é igual ao campo elétrico vezes a carga inserida e que a carga de um próton é de 1.6 vezes 10 a -19 Coulombs positivo. Se temos o módulo do campo elétrico e temos a carga, resolver a força é bem fácil:
$$F = (40N/C) * 1.6*10^{-19}C = 64*10^{-19}N$$
Para a B, é só saber que, no campo elétrico, quanto maior a distância entre as linhas de campo, menor é seu módulo. Como a distância entre as linhas de campo no ponto B é o dobro da distância de no ponto A:
$$E_b = \dfrac{E_a}{2} = \dfrac{40N/C}{2} = 20N/C$$
Resolvido.

5. O núcleo de um átomo de plutônio-239 contém 94 prótons. Suponha que o núcleo é uma esfera com 6.64fm de raio e que a carga dos prótons está distribuída uniformemente nessa esfera. Determine:
a) o módulo;
b) o sentido (para dentro ou para fora) do campo elétrico produzido pelos prótons na superfície do núcleo.
Primeiro, vamos analisar as informações que temos:
$$q = 94*1.6*10^{-19}C = 150.4*10^{-19}C \\
r = 6.64*10^{-15}m$$
No caso, a carga do plutônio é a carga contida em 94 prótons, e o raio da esfera é igual a 6.64 vezes 10 a -15 metros.

Para determinar o módulo do campo elétrico, temos a seguinte equação:
$$E = \dfrac{1}{4 \pi \varepsilon_0} * \dfrac{q}{d^2} = 8.99*10^9 Nm^2/C^2 * \dfrac{150.4*10^{-19}C}{6.64*10^{-15}m}^2 = \dfrac{1352.096*10^{-10} Nm^2/C}{44.0896*10^{-30}m^2} \\
 \approx 30.667*10^{20} N/C$$
Bem-respondida está a letra A.

Quanto a letra B, não tem o que equacionar. A carga do próton é positiva, então o campo elétrico produzido pelos prótons é divergente, ou seja, para fora.

6. Duas partículas são mantidas fixas sobre o eixo x: a partícula 1, de carga q1 no ponto x = 6.00cm, e a partícula q2 no ponto x = 21.0cm. Qual é o campo elétrico total a meio caminho entre as partículas, em termos dos vetores unitários?
$$q_1 = -2.00*10^{-7}C \\
q_2 = +2.00*10^{-7}C \\
x_1 = 6.00cm = 0.06m \\
x_2 = 21.0cm = 0.21m$$
Outro exercício bem fácil, mas pega na questão da distância, de maneira um pouco diferente daqueles exercícios do capítulo 21. A distância é a seguinte: você vai fixar o campo elétrico num ponto entre a primeira partícula e a segunda, e a distância será entre esse ponto e o x da partícula.
Primeiro, o ponto entre a primeira e a segunda:
$$x_E = \dfrac{0.21m + 0.06m}{2} = \dfrac{0.27m}{2} = 0.135m$$
Depois, as distâncias (que logicamente tem de ter módulo igual, mas sinais opostos):
$$d_1 = 0.135m - 0.06m = 0.075m\\
d_2 = 0.135m - 0.21m = -0.075m$$
Agora, se o campo elétrico total é a soma do campo com relação à primeira partícula e com relação à segunda:
$$E_T = E_1 + E_2 = K \dfrac{|q_1|}{d_1 ^2} + K \dfrac{|q_2|}{d_2 ^2} = K (\dfrac{|2*10^{-7}C|}{(0.075m)^2} + {|2*10^{-7}C|}{(-0.075m)^2}\\
= K (\dfrac{2*10^{-7}C}{0.005625m^2} + \dfrac{2*10^{-7}C}{0.005625m^2}) = K \dfrac{4*10^{-7}C}{0.005625m^2} \\
\approx 8.99*10^9 Nm^2/C^2 * 711.11*10^{-7}C/m^2 = 6392.8789*10^2 N/C$$

Se for pra expressar em termos dos vetores unitários, bem, o exercício especifica que as partículas são mantidas fixas no eixo x, aonde y = 0. Se o campo trabalha completamente no eixo x, o ângulo trabalhado é 0 - a equação no eixo x será multiplicada por cosseno de 0, e no eixo y pelo seno de 0.
Logo, ele é equivalente ao módulo em x e, por consequência, igual a 0 em y.

10. Na figura, as quatro partículas são mantidas fixas e têm cargas q1 = q2 = +5e, q3 = +3e e q4 = -12e. A distância d = 5.0 micrômetros. Qual é o módulo do campo elétrico no ponto P?
Esse exercício é facílimo de interpretar e dá pra responder sem fazer nenhuma conta, apenas usando deduções, mas se quiser usar as equações fica um pouquinho mais complicado. Não muito, de qualquer forma. Queremos saber o campo resultante dos 4 campos com relação ao ponto P, então o ponto de partida é esse:
$$E = E_1 + E_2 + E_3 + E_4$$
Uma informação que podemos ver pelo gráfico é que, com relação ao ponto P, a primeira carga está oposta à segunda, então vamos estabelecer a seguinte relação:
$$E_1 = -E_2$$
Também é bom lembrar que a distância entre P e q4 não é d, e sim 2d, como podemos notar pelo próprio gráfico. E que, com relação a P, um dos campos elétricos têm de ter carga negativa, já que a carga 3 é positiva e a carga 4 é negativa. Agora é só terminar o trabalho com a equação:
$$E = E_1 - E_1 + E_3 + E_4 = E_3 + E_4 = -K \dfrac{q_3}{d_3 ^2} + K \dfrac{q_4}{d_4 ^2}] = K (- \dfrac{3e}{d^2} + \dfrac{12e}{(2d)^2}) \\
K (-\dfrac{3e}{d^2} + \dfrac{12e}{4d^2}) = K * 0 = 0$$
Logo, o módulo do campo elétrico no ponto P é 0. Isso é facilmente explicado: todas as linhas de campo são na direção de 4, a única carga oposta, e como nenhuma delas é forte o suficiente para tocar o campo P "por acaso", o campo em P é nulo.

13. Na figura, as três partículas são mantidas fixas no lugar e têm cargas q1 = q2 = +e e q3 = +2e. A distância a = 6.00 micrômetros. Determine:
a) o módulo e b) a direção do campo elétrico no ponto P.
Outro exercício semelhante a um que maior parte patinou na prova, e de fato era chatinho, em especial pela questão da distância, mas nem tão problemático assim. Temos um ponto de partida extremamente semelhante ao exercício passado porque, se você perceber bem, com relação ao ponto P a primeira e a segunda carga se anulam. Exatamente porque as duas têm cargas iguais e o ponto P está entre elas.
Visto isso, notamos que o campo elétrico em P está integralmente relacionado ao campo elétrico da terceira carga. Logo:
$$E_P = E_3$$
Temos que a carga de 3 corresponde a dois elétrons, que resulta em:
$$q_3 = 2*1.6*10^{-19}C = 3.2*10^{-19}C$$
E o problema maior é a distância, mas podemos facilmente fazer essa relação notando que P está entre a primeira e a segunda carga, que estão em eixos opostos no plano cartesiano. O P está no meio da hipotenusa de um triângulo fechado pelas cargas q1 e q2. Se cada cateto é a distância a (6 micrômetros), temos que a hipotenusa é:
$$h = \sqrt{6^2 + 6^2} = \sqrt{36 + 36} = \sqrt{72} \mu m$$
Ótimo, temos a hipotenusa... Mas algo que pegou muita gente é que a distância entre q3 e o ponto P não é a hipotenusa. O ponto P está no centro dessa hipotenusa, ou seja:
$$d = \dfrac{\sqrt{72}}{2} \mu m = \dfrac{\sqrt{72}}{2}*10^{-6}m$$
(lembrando que não aproximei o resultado porque elevaremos ele ao quadrado e podemos descobrir algo mais exato)
Com essa distância, temos tudo o que precisamos para descobrir o módulo do campo elétrico:
$$E = K \dfrac{q}{d^2} = 8.99*10^9 Nm^2/C^2 * \dfrac{3.2*10^{-19}C}{(\dfrac{\sqrt{72}}{2} * 10^{-6}m)^2} = \dfrac{28.768*10^{-10} Nm^2/C}{\dfrac{72}{4} * 10^{-12}m^2} \\
\dfrac{28.768*4*10^2 N/C}{72} \approx 1.598*10^2 N/C \approx 160N/C$$
E essa é a resposta. Parece complicadinho, mas depois de deixar de estriar com a distância o exercício fica praticamente pronto.

Quanto ao ângulo (direção), é fácil de deduzir. Ainda no triângulo q1-q2-q3, temos um triângulo cujos dois catetos são iguais, logo têm dois ângulos de 45 graus e um de 90 - esse, no caso, é q3. Mas não estamos tratando desse triângulo, estamos tratando de um triângulo q1-q3-P ou q2-q3-P.
Não seria difícil fazer essa relação geometricamente falando, mas o exercício nos dá uma dica valiosíssima que não nos deixa perder tempo: o ângulo no ponto P do triângulo q2-q3-P é 90°. Se q2 continua tendo 45°, q3 nessa ocasião só pode ter 45° também.
Logo, a direção desse campo elétrico é de 45°.


16. A figura mostra um anel de plástico de raio R = 50.0cm. Duas pequenas contas coloridas estão sobre um anel: a conta 1, de carga +2.00 microC, que é mantida fixa na extremidade esquerda, e a conta 2, de carga +6.00 microC, que pode ser deslocada ao longo do anel. As duas contas, juntas, produzem um campo elétrico de módulo E no centro do anel. Determine:
a) um valor positivo do ângulo para o dado valor de E e b) um valor negativo.
$$E = 2.00*10^5 N/C \\
q_1 = +2.00 \mu C = +2.00*10^{-6} C \\
q_2 = +6.00 \mu C = +6.00*10^{-6} C \\
R = 50cm = 0.5m$$
Esse exercício é outro que precisa de umas sacadinhas de trigonometria pra resolver, mas não é nada realmente preocupante. Veja bem, temos um campo elétrico total que nos foi dado, mas sabemos que esse campo total é o que fica entre a primeira e a segunda conta. Ou seja, ele leva em conta tanto a primeira conta quanto a segunda.
$$E = E_1 + E_2$$
Mas lembremos que não podemos simplesmente somar os campos elétricos da primeira e da segunda conta para descobrir o que está no centro. Perceba que eles são grandezas vetoriais e, como tais, têm seu próprio sentido - o campo elétrico da primeira "aponta" para frente com relação ao centro, enquanto o segundo aponta para trás e para baixo numa direção desconhecida (que, inclusive, é o que queremos saber); sendo assim, seu sentido é negativo.
Vamos montar as equações de cada campo em cada direção pra exemplificar melhor:
$$E_1 = (\dfrac{1}{4 \pi \varepsilon_0} * \dfrac{q_1}{R^2} \cos{0})\hat{i} + (\dfrac{1}{4 \pi \varepsilon_0} * \dfrac{q_1}{R^2} \sin{0})\hat{j} = (\dfrac{1}{4 \pi \varepsilon_0} * \dfrac{q_1}{R^2})\hat{i} \\
E_2 = -(\dfrac{1}{4 \pi \varepsilon_0} * \dfrac{q_2}{R^2} \cos{\theta})\hat{i} - (\dfrac{1}{4 \pi \varepsilon_0} * \dfrac{q_2}{R^2} \sin{\theta})\hat{j}$$
Perceba que já defini o ângulo do campo da primeira conta como 0, afinal, ela está no eixo x.

Agora o que faremos é uma soma disso:
$$E = E_1 + E_2 = ((\dfrac{1}{4 \pi \varepsilon_0} * \dfrac{q_1}{R^2})\hat{i}) + (-(\dfrac{1}{4 \pi \varepsilon_0} * \dfrac{q_2}{R^2} \cos{\theta})\hat{i} - (\dfrac{1}{4 \pi \varepsilon_0} * \dfrac{q_2}{R^2} \sin{\theta})\hat{j})$$
Vamos separar os i dos j:
$$E = (\dfrac{1}{4 \pi \varepsilon_0} * \dfrac{q_1}{R^2} - \dfrac{q_2}{R^2} \cos{\theta}) - (\dfrac{1}{4 \pi \varepsilon_0} * \dfrac{q_2}{R^2} \sin{\theta})\hat{j})$$
Agora temos que fazer uma análise trigonométrica do caso: queremos o ângulo theta, mas estamos com equações vetoriais e precisamos convertê-las para que fique escalar. Normalmente usaríamos o teorema de Pitágoras, mas o próprio ângulo é uma incógnita e... Inclusive a que queremos saber. Convém-nos então utilizar a lei dos cossenos (ou teorema de Pitágoras level 2 pros que gostam de associar):
$$a^2 = b^2 + c^2 - 2ab \cos{\theta}$$
Substituindo os valores por uma hipotenusa E, uma direção Ei e outra direção Ej:
$$E^2 = (K \dfrac{q_1}{R^2})^2 + (K \dfrac{q_2}{R^2})^2 - 2(K \dfrac{q_1}{R^2})(K \dfrac{q_2}{R^2}) \cos{\theta} \\
E^2 = K^2 \dfrac{q_1 ^2}{(R^2)^2} + K^2 \dfrac{q_2 ^2}{(R^2)^2} - 2 K^2 \dfrac{q_1 * q_2}{(R^2)^2} \cos{\theta} \\
E^2 = K^2 \dfrac{q_1 ^2}{R^4} + K^2 \dfrac{q_2 ^2}{R^4} - 2 K^2 \dfrac{q_1 * q_2}{R^4} \cos{\theta} \\
E^2 = \dfrac{K^2}{R^4} (q_1 ^2 + q_2 ^2 - 2 q_1 q_2 \cos{\theta})$$
Agora vamos isolar quem realmente interessa: quem está com o theta.
$$E + \dfrac{K^2}{R^4}*(2 q_1 q_2 \cos{\theta}) = \dfrac{K^2}{R^4} (q_1 ^2 + q_2 ^2) \\
\dfrac{K^2}{R^4} * (2 q_1 q_2 \cos{\theta}) = \dfrac{K^2}{R^4} (q_1 ^2 + q_2 ^2) - E^2 \\
2 q_1 q_2 \cos{\theta} = \dfrac{K^2 * R^4}{R^4 * K^2} (q_1 ^2 + q_2 ^2) - \dfrac{R^4}{K^2} E^2 \\
2 q_1 q_2 \cos{\theta} = q_1 ^2 + q_2 ^2 - \dfrac{R^4}{K^2} E^2 \\
\cos{\theta} = \dfrac{q_1 ^2 + q_2 ^2 - \dfrac{R^4}{K^2} E^2}{2 q_1 q_2} \\
\theta = \cos^{-1}{\dfrac{q_1 ^2 + q_2 ^2 - \dfrac{R^4}{K^2} E^2}{2 q_1 q_2}}$$
Agora substitua as variáveis dessa equação pelos valores dados pelo exercício na sua companheira calculadora científica. Ou no seu Matlab. A conta é extremamente extensa e cansativa, mas é algo que a calculadora pode fazer sem nenhum trabalho.
A resposta tem que dar aproximadamente 67.8°.

Quanto ao ângulo negativo... Ele será, bem, -67.8° mesmo.

19. A figura mostra um dipolo elétrico. Determine:
a) o módulo;
b) a orientação (em relação ao semi-eixo x positivo) do campo elétrico produzido pelo dipolo em um ponto P situado a uma distância r >> d.

É um exercício de raciocínio simples. O campo elétrico está no centro e, como tal, tem influência do campo gerado pela carga positiva e pela carga negativa. E, claro, como uma carga está divergindo e outra está convergindo, elas não se anulam e sim se acumulam num ponto só. Essa é a sacada do dipolo, que aliás é simplíssimo - o bom é que não é dado valor nenhum, então não há resultado constante a se considerar, é pra chegar numa equação de dipolo genérica.
Mas veja bem, como eu disse: as cargas não se acumulam, elas se juntam. Só com isso já dá pra imaginar o resultado do campo elétrico... Mas façamos uma análise mais minuciosa, pra não pecar em não entender o exercício como um todo.
Note que as duas cargas estão no mesmo eixo y, de ângulo desconhecido, mas possivelmente ignorando o eixo x. Como dito, elas se "complementam", então estão no mesmo sentido. Com isso concluímos que:
$$E_+ = E_{+y}\hat{j} = E \sin{\theta} \hat{j} \\ E_- = E_{-y}\hat{j} =  E \sin{\theta} \hat{j} = E \hat{j} \\ E_{+x} = E_{-x} \\ |\vec{E}| = E_+ + E_- = E\hat{j} + E\hat{j} = 2E \sin{\theta}$$
Até aqui ok, vem agora outra parte: definir a equação completa pro campo. Talvez a pior.
Lembremos que a equação básica do campo elétrico é:
$$E = K \dfrac{q}{d^2}$$
Não temos o valor de q então ele ficará no resultado final, enquanto a distância d pode ser descoberta através da seguinte relação:
Ainda não defini a resultante porque se não dá aquele problema do cara querer chegar na resposta final direto e whatever. O fato é: veja que formamos triângulos retângulos entre o eixo x e y e essas linhas, logo essas linhas são hipotenusas. Podemos considerar o eixo x desse campo a variável r, enquanto o eixo y é representado por d/2. Pelo teorema de Pitágoras:
$$d_{E} = \sqrt{(\dfrac{d}{2})^2 + r^2}$$
Jogando na equação já final:
$$|\vec{E}| = 2(K \dfrac{q}{(\sqrt{(\dfrac{d}{2})^2 + r^2})^2}) \sin{\theta} = 2(K \dfrac{q}{(\dfrac{d}{2})^2 + r^2}) \sin{\theta}$$
Quanto ao seno de theta, vamos fazer relações trigonométricas básicas do ensino médio. O básico, seno é igual ao cateto oposto sobre a hipotenusa. De uma série de maneiras, uma delas sendo o fato de saber que d/2 está relacionado com o seno (quanto mais perto de 90° o ângulo, mais perto o vetor chega de d/2 absoluto), podemos definir o cateto oposto como d/2. A hipotenusa acabamos de expressar. Logo, no lugar de seno colocamos:
$$\sin{\theta} = \dfrac{\dfrac{d}{2}}{\sqrt{(\dfrac{d}{2})^2 + r^2}}$$
E jogamos na equação final:
$$|\vec{E}| = 2 \times (K \dfrac{q}{(\dfrac{d}{2})^2 + r^2}) \times \dfrac{\dfrac{d}{2}}{\sqrt{(\dfrac{d}{2})^2 + r^2}} = \dfrac{2Kqd}{2 \sqrt{((\dfrac{d}{2})^2 + r^2)^3}} = \dfrac{Kqd}{ \sqrt{((\dfrac{d}{2})^2 + r^2)^3}}$$
Sabemos que r >> d, logo, acrescentar qualquer valor de d para r se torna razoavelmente insignificante, então podemos simplesmente descartá-lo e deixar a equação dessa maneira:
$$|\vec{E}| = \dfrac{Kqd}{(\sqrt{r^2})^3} = \dfrac{Kqd}{r^3}$$
E está respondido o primeiro.

O segundo acabamos considerando a resposta no meio do raciocínio do primeiro. Em algum momento, dissemos da relação de d/2 com o seno e concluímos que ele tem 90°. Lembrando que as linhas vão em sentido à carga negativa, então o ângulo correto é -90°.

25. Na figura, duas barras curvas de plástico, uma de carga +q e outra de carga -q, formam uma circunferência de raio R = 8.50cm no plano xy. O eixo x passa pelos dois pontos de ligação entre os arcos, e a carga está distribuída uniformemente nos dois arcos. Se q = 15.0 picoC, determine:
a) o módulo;
b) a orientação (em relação ao semi-eixo x positivo) do campo elétrico E no ponto P, situado no centro da circunferência.

Galera, eu tinha uma resposta quase certa mas fracassei num detalhe que compromete toda a conta... No caso, um pi³ no lugar de um pi² no divisor. De qualquer forma, a expressão usada por uma resolução automática é decompor a carga dessa maneira:
$$q = \lambda \pi R$$
E jogar na equação do campo elétrico:
$$|\vec{E}| = E_+ + E_- = 2E \sin{\theta}$$
(até aí, igualzinho ao exercício anterior)
$$|\vec{E}| = 2 \dfrac{1}{4 \pi \varepsilon_0} \times \dfrac{\lambda \pi R}{\pi R^2} \sin{\theta} = \dfrac{\lambda \pi R}{2R^2 \pi^2 \varepsilon_0} (\sin{-90} - \sin{90}) = \\ \dfrac{\lambda}{2R \pi \varepsilon_0} \times (-2) = -\dfrac{q}{\pi^2 \varepsilon_0 R^2}$$
Daí jogando os valores:
$$|\vec{E}| = -\dfrac{15 \times 10^{-12}}{\pi^2 \varepsilon_0 (0.085)^2} = -\dfrac{15 \times 10^{-12}C}{8.85 \times 10^{-12} \times \pi^2 \times 0.007225} \\ = -\dfrac{15}{0.06394125 \pi^2} \approx \dfrac{234.59}{\pi^2} \approx 23.77N/C$$
E é isso aí. Deve ter reconhecido um problema no piR², ou está certo e eu estou confuso na matemática básica, sei lá. Mas o resultado é esse.

Quanto ao ângulo, perceba que a carga negativa está na parte inferior do círculo. É a mesma associação do exercício passado, -90°. Ao menos isso eu entendi bem, rs. Perdão pelo exercício, galera. Próximo.

30. Uma barra fina não-condutora, com uma distribuição uniforme de carga positiva Q, tem a forma de um círculo de raio R. O eixo central do anel é o eixo z, com a origem no centro do anel. Determine o módulo do campo elétrico:
a) no ponto z = 0;
b) no ponto z tendendo a infinito;
c) em termos de R, para que valor positivo de z o módulo do campo é máximo?
d) se R = 2.00cm e Q = 4.00 microC, qual é o valor máximo do campo?
Ok, essa questão é chata... Acho que caiu na primeira prova, até. Aqui o esquema é o seguinte: podemos fazer uma análise por equações prontas que o livro do Halliday nos ensina. Podemos fazer aquela análise por integral também. Vou fazer o completo (aproveitar que o livro tem tudo rs) mas a parte importante mesmo é só a equação final.
Veja bem:
$$\cos{\theta} = \dfrac{z}{r} = \dfrac{z}{(z^2 + R^2)^{1/2}}$$
(o r menor virar aquilo é devido ao fato do r menor ser a hipotenusa de um triângulo retângulo formado pelos catetos z e R)
Sendo:
$$dE = K \dfrac{\lambda ds}{(z^2 + R^2)}$$
Juntamos dE com o cosseno:
$$dE \cos{\theta} = K \lambda \dfrac{z}{(z^2 + R^2)^{3/2}} ds$$
Integramos os dois lados (s sendo o arco do anel, indo de 0 a 2piR):
$$E = \int dE \cos{\theta} = \int K \lambda \dfrac{z}{(z^2 + R^2)^{3/2}} ds = K \lambda \dfrac{z}{(z^2 + R^2)^{3/2}} 2 \pi R$$
O lambda com o 2piR forma uma carga q, o resto permanece igual:
$$E = \dfrac{Kqz}{(z^2 + R^2)^{3/2}}$$

Essa é a equação fixa do anel. Para o exercício A, precisamos substituir z por 0.
$$E = \dfrac{Kq}{(0 + R^2)^{3/2}} \times 0 = 0$$

Para o exercício B, jogamos um limite de z tendendo a infinito após trabalhar a equação:
$$E = \lim_{z\to\infty} Kq \dfrac{z}{(z^2 + R^2)^{3/2}}$$
Dá pra escapar daqui por um monte de método, mas no blog é bom economizar espaço. Vamos então aplicar o teorema de L'Hopital (aquele do cálculo I, lembram?), derivar em cima e embaixo com relação a z pra facilitar as coisas:
$$E = \lim_{z\to\infty} Kq \dfrac{1}{2z \times \dfrac{3}{2} (z^2 + R^2)^{1/2}} = \lim_{z\to\infty} Kq \dfrac{1}{3z \times (z^2 + R^2)^{1/2}}$$
Jogue os limites, o resultado tende a 0. Grato ao arroba @armiquilino (Alexandre Miquilino) por me lembrar do L'Hopital. (começando as participações especiais no blog!)

Para o exercício C, queremos o valor de z para o campo máximo. A conclusão a se chegar é fácil, o único chato é fazer mesmo. Quando queremos o valor máximo de alguma coisa usando alguma variável, já nos vem de cara setar a derivada da função incógnita com relação à variável a 0. Assim, em prática:
$$\dfrac{df(x)}{dx} = 0$$
No caso:
$$\dfrac{df(z)}{dz} = \dfrac{d}{dz} [Kq \dfrac{z}{(z^2 + R^2)^{3/2}}] = Kq \dfrac{d}{dz} [z \times (z^2 + R^2)^{-3/2}] = \\ Kq [(1 \times (z^2 + R^2)^{-3/2}) + (z \times (2z \times \dfrac{-3}{2} (z^2 + R^2)^{-5/2}))] = \\ Kq [\dfrac{1}{(z^2 + R^2)^{3/2}} - \dfrac{3z^2}{(z^2 + R^2)^{5/2}}] = \\ Kq [\dfrac{(z^2 + R^2) - 3z^2}{(z^2 + R^2)^{5/2}}] = Kq \dfrac{R^2 - 2z^2}{(z^2 + R^2)^{5/2}} = 0$$
Agora queremos isolar o z, não é? Vamos fazer o seguinte então. Note que a expressão acima pode ser vista também como:
$$Kq \dfrac{R^2}{(z^2 + R^2)^{5/2}} - Kq \dfrac{2z^2}{(z^2 + R^2)^{5/2}} = 0$$
Passando a parte do -2z pro outro lado como positivo:
$$Kq \dfrac{2z^2}{(z^2 + R^2)^{5/2}} = Kq \dfrac{R^2}{(z^2 + R^2)^{5/2}}$$
Cortando as partes iguais dos dois lados:
$$2z^2 = R^2 \\ z^2 = \dfrac{R^2}{2} \\ z = \sqrt{\dfrac{R^2}{2}} = \dfrac{R}{\sqrt{2}} \approx 0.707R$$
Isso, claro, porque queremos o valor positivo de z. Poderia ser -0.707R também por causa da raiz.
Bem trabalhoso, não é?

Para o exercício D, temos dois valores dados, a carga C (4 microC) e o raio R (2cm). O valor de z é o máximo, logo vamos substituir z pelo valor descoberto no exercício passado. Só jogar tudo na equação do anel:
$$E = Kq\dfrac{z}{(z^2 + R^2)^{3/2}} = (8.99 \times 10^9 Nm^2/C^2) \times (4 \times 10^{-6} C) \times \dfrac{\dfrac{R}{\sqrt{2}}}{((\dfrac{R}{\sqrt{2}})^2 + (0.02m)^2)^{3/2}} = \\ 35.96 \times 10^3 Nm^2/C \times \dfrac{\dfrac{0.02m}{\sqrt{2}}}{((\dfrac{0.02m}{\sqrt{2}})^2 + 0.0004m^2)^{3/2}} = \\ 35.96 \times 10^3 Nm^2/C \times \dfrac{0.02m}{\sqrt{2} \times ((\dfrac{0.0004m^2}{2}) + 0.0004m^2)^{3/2}} = \\ 35.96 \times 10^3 Nm^2/C \times \dfrac{0.02m}{\sqrt{2} \times (0.0002m^2 + 0.0004m^2)^{3/2}} = \\ 35.96 \times 10^3 Nm^2/C \times \dfrac{0.02m}{\sqrt{2} \times (0.0006m^2)^{3/2}} \approx \\ 35.96 \times 10^3 Nm^2/C \times \dfrac{0.02m}{\sqrt{2} \times (1.47 \times 10^{-5})} = \\ 35.96 \times 10^3 Nm^2/C \dfrac{1360.83}{\sqrt{2} m^2} = \\ \dfrac{48935 \times 10^3 Nm^2/C}{\sqrt{2} m^2} \approx 34602.59 \times 10^3 N/C \approx 3.5 \times 10^7 N/C$$
Enfim está encerrado. Acho que esse 30 é o mais escroto do capítulo 22. Depois disso é tudo mais pacífico.

O resto eu não vou fazer por motivos de: já passou o semestre e eu nem tenho mais as coisas aqui, e o exame já foi e etc. Então fiquem com esses exercícios mesmo, talvez sejam úteis. :Ç

Queda Livre para Exame - Cálculo Diferencial e Integral III

Ok, me ofereceram uma bolada de 150 dólares pra esquecer um pouco as férias e curtir cálculo III com a galera que ficou de exame, então aqui estou eu. O pedido foi queda livre, então peguei os... três exercícios do livro sobre queda livre pra resolver aqui detalhadamente pra vocês.
Como de costume, ressaltarei no final. Mas estudem bastante o primeiro, o segundo é deveras desnecessário do ponto de vista de uma prova de cálculo, e o terceiro será apenas no caso de ele querer usar queda livre como exam killer.

1. Deixa-se cair um corpo de 10 "slugs" de massa de uma altura de 1000 pés sem velocidade inicial. A resistência do ar é proporcional à velocidade do corpo. Se a velocidade limite é 320 pés/s, determine:
a) uma expressão para a velocidade no instante t;
b) o tempo necessário para o corpo atingir a velocidade de 160 pés/s.
Ok, galera. Queda livre é um negócio menos complicado do que parece. Vamos começar lá do princípio, puxar a equação pelo esqueleto dela, e aí começamos a trabalhar porque aí dá pra entender bem. A equação usada aqui é:

$$F_g = ma + Kv$$
Sendo Fg a força da gravidade, a massa do objeto, a a aceleração, uma pequena proporcionalidade da resistência do ar e v a velocidade. Podemos igualar F a mg, ficando assim:
$$mg = ma + Kv$$
Passamos m dividindo tudo:
$$g = a + K \dfrac{v}{m}$$
Ok, mais uma consideração: quando a resistência do ar é proporcional à velocidade do corpo, temos que aquela constante K é definida pela razão entre o peso (massa vezes aceleração da gravidade) e a velocidade inicial. Difícil de entender? Seguinte:
$$K = \dfrac{mg}{v_0}$$
Sempre quando o exercício disser "A resistência do ar é proporcional à velocidade do corpo".
Substituindo isso na equação temos:
$$g = a + \dfrac{mg}{v_0} \times \dfrac{v}{m} = a + \dfrac{gv}{v_0}$$
Sendo assim, a equação está quase pronta pra gente trabalhar nela. Falta uma única consideração antes de jogar os valores constantes: note que a aceleração a é a derivada da velocidade com relação ao tempo, logo:
$$\dfrac{dv}{dt} + \dfrac{gv}{v_0} = g$$
Agora sim temos uma equação diferencial bonita e gostosa de se trabalhar. Ao jogar os valores, só precisamos fazer uma consideração: no sistema de medidas utilizado no exercício (slugs, pés, etc.) a gravidade se dá por 32.2 pés/s², que o exercício nos dá a colher de chá pra arrendondar pra 32 pés/s². Logo, jogamos tudo o que o exercício nos dá:
$$\dfrac{dv}{dt} + \dfrac{32}{320} v = 32 \\ \dfrac{dv}{dt} + 0.1v = 32$$
Resolvendo a equação diferencial usando o método do fator integrante:
$$I(t, v) = e^{\int 0.1 dt} = e^{0.1t}$$
$$e^{0.1t} [\dfrac{dv}{dt} + 0.1v] = 32e^{0.1t} \\ \dfrac{d}{dt} [ve^{0.1t}] = 32e^{0.1t}$$
Integrando dos dois lados:
$$ve^{0.1t} = 32 \dfrac{e^{0.1t}}{0.1} + C = 320e^{0.1t} + C$$
Isolando v:
$$v = 320 + Ce^{-0.1t}$$

Agora vamos analisar as informações que o exercício nos dá pra determinar a constante C: ele nos diz que não há velocidade inicial, o que indica que a velocidade inicial é 0. Logo, quando t = 0, v = 0. Podemos usar isso na equação:
$$v(0) = 320 + Ce^0 = 0 \\ 320 + C = 0 \\ C = -320$$
Sendo assim:
$$v = 320 - 320e^{-0.1t}$$

Essa é a resposta da letra a, que pede uma expressão para a velocidade no instante t. Como vê, a única variável da equação é t, o que indica que chegamos no melhor estado possível pra equação.
Tendo essa equação, no entanto, fica fácil resolver a letra b, que pede o tempo necessário para o corpo atingir 160 pés/s. Só substituir v por 160 e isolar t. Veja:
$$V(t) = 320 - 320^{-0.1t} = 160 \\ 320e^{-0.1t} = 320 - 160 = 160 \\ e^{-0.1t} = 0.5 \\ -0.1t = \ln{0.5} \approx -0.693 \\ t = \dfrac{-0.693}{-0.1} = 6.93$$
Ou seja, o tempo necessário é de 6.93 segundos. Resolvido o primeiro exercício.

2. Lança-se um corpo de massa m verticalmente para cima com velocidade inicial v0. Supondo nula resistência do ar, determine:
a) a equação do movimento no sistema de coordenadas da figura;
b) uma expressão para a velocidade do corpo no instante t;
c) o instante tm em que o corpo atinge altura máxima;
d) uma expressão para a posição do corpo no instante t;
e) a altura máxima atingida pelo corpo.
Vamos reconsiderar a equação que elaboramos anteriormente:

$$\dfrac{dv}{dt} + \dfrac{Kv}{m} = g$$
Veja bem, o exercício nos dá uma ajudinha aqui: "supondo nula resistência do ar". Resistência do ar nula indica que a constante K é 0. E outra consideração relevante é o fato do corpo estar sendo lançado verticalmente para cima, sendo assim a gravidade se opõe a ele e numa análise vetorial mais complexa chegaríamos à inversão de sinais de g.
Mas olha, vamos apenas usar a lógica básica: no outro exercício o objeto tava descendo então g era positivo, nesse ele está subindo então g é negativo. Simples assim.
Logo:
$$\dfrac{dv}{dt} = -g$$

Essa é a resposta pra letra a. Para as outras vamos chegar através de integrais a todas as equações que nos jogaram no ensino médio. Veja bem, a letra b pede a velocidade do corpo no instante t. Se temos dv/dt, só integrar:
$$v = \int \dfrac{dv}{dt} dt = \int -g \\ v = -gt + C$$
Temos que a velocidade inicial (t = 0) é v0, então façamos essa substituição:
$$v(0) = C = v_0$$
Sendo assim:
$$v = v_0 - gt$$

Para determinar o instante tm aonde a altura é máxima, temos que usar aquele princípio básico de equação de segundo grau. Lembra? Máximos e mínimos. O valor máximo de uma parábola com concavidade pra baixo é descoberto ao igualar a derivada da função a 0. A função, no caso, é a posição. Ou seja:
$$\dfrac{ds}{dt} = 0$$
Sabemos pelas leis fundamentais da mecânica que a velocidade é a derivada da posição, logo usamos a equação que descobrimos anteriormente:
$$\dfrac{ds}{dt} = v_0 - gt \\ v_0 - gt = 0$$
Agora vamos fazer o seguinte: já que não temos valores constantes pra colocar aí, vamos isolar t e criar uma equação pra isso. É suficiente.
$$gt = v_0 \\ t = \dfrac{v_0}{g}$$

O outro exercício nos pede uma expressão para posição. Raciocínio lógico puro. Se a velocidade é a derivada da posição, a posição é a integral da velocidade, sendo assim vamos integrar aquela equação que achamos para a letra b.
$$S = \int (v_0 - gt) dt = v_0 t - g(\dfrac{1}{2} t^2) + C = v_0 t - \dfrac{gt^2}{2} + C$$
Temos que a posição inicial é 0. Logo, (t = 0, v = 0). Jogando os valores:
$$S(0) = 0v_0 - \dfrac{0g}{2} + C = 0 \\ C = 0$$
E a equação final é:
$$S = V_0 t - \dfrac{gt^2}{2}$$

E o último pede a altura máxima, não o tempo necessário pra ela. Mas bem, se temos o tempo necessário pra ela, é só substituir t pelo tm que descobrimos na letra c.
$$S_{max} = V_0 \dfrac{V_0}{g} - \dfrac{g}{2} (\dfrac{V_0}{g})^2 = \dfrac{V_0^2}{g} - \dfrac{gV_0^2}{2g^2} = \dfrac{V_0^2}{g} - \dfrac{V_0^2}{2g} = \dfrac{V_0^2}{2g}$$

E enfim está tudo resolvido. Parece meio confuso a parte de máximo e tal, mas duvido bastante que isso vá cair na prova, é só pra ter queda livre dissecada passo a passo.

3. Um corpo de 1 "slug" de massa é solto no espaço com velocidade inicial de 1 pé/s, e encontra uma resistência do ar dada exatamente por -8v². Determine a velocidade no instante t.
Parece o exercício mais fácil, mas é muito mais difícil que o primeiro também. Devido ao fato do corpo ser lançado no espaço, temos que estabelecer uma força com relação ao corpo, e não à gravidade. Assim:
$$F_c = F_g - 8v^2 \\ ma = mg - 8v^2 \\ m \dfrac{dv}{dt} = mg - 8v^2$$
Substituindo os valores que temos:
$$1 \dfrac{dv}{dt} = 1 \times 32 - 8v^2 = 32 - 8v^2$$
Ok, e agora? Fator integrante? Não. Não temos a equação no formato de fator integrante, o que vamos fazer é "brincar" com ela um pouquinho com aquele método distributivo que aprendemos lá no começo.
$$dv = (32 - 8v^2)dt \\ \dfrac{dv}{32 - 8v^2} = dt \\ \dfrac{dv}{32 - 8v^2} - dt = 0$$
Integrando toda a equação temos:
$$\int \dfrac{1}{32 - 8v^2} dv - \int dt = C$$
A segunda é fácil, a primeira vamos usar a ajuda das nossas boas calculadoras porque usa um método horrível. É muito duvidoso que caia algo do tipo na prova, então é só pra passar isso logo e ir pra parte que interessa mais: o resultado.
$$(\dfrac{1}{32} [\ln{|v + 2|} - \ln{|2 - v|}]) - t = C$$
Usando mais propriedades (lnA - lnB = lnA/lnB) pra simplificar o que queremos:
$$\dfrac{1}{32} [\ln{|\dfrac{2 + v}{2 - v}|}] = C + t \\ \ln{|\dfrac{2 + v}{2 - v}|} = 32C + 32t = K + 32t \\ \dfrac{2 + v}{2 - v} = e^{K + 32t}$$
Uma das respostas do livro chega nesse formato. A outra é v isolado bonitinho. Quem quiser isolar completamente v, tem essa saída brincando com álgebra até cansar (vou chamar convenientemente K + 32t de x).
A princípio, jogamos o 2 pro outro lado:
$$\dfrac{v}{2 - v} = e^x - \dfrac{2}{2 - v}$$
Depois multiplicamos os dois lados por 2-v pra deixar v sozinho:
$$v = e^x (2 - v) - \dfrac{2(2 - v)}{2 - v} = (2e^x) - ve^x - 2$$
Então passamos ve^x pro outro lado de novo, porque vamos fazer um jogo de fatoração bem interessante:
$$v + ve^x = 2e^x - 2$$
Fatorando:
$$v(1 + e^x) = 2e^x - 2$$
Isolando v:
$$v = \dfrac{2e^x - 2}{e^x + 1} = 2 \times \dfrac{e^{K + 32t} - 1}{e^{K + 32t} + 1}$$
Agora, claro, vamos descobrir o valor de K. Que aliás usaremos a seguinte propriedade:
$$e^{K + 32t} = e^K \times e^{32t} = xe^{32t}$$
Ou seja: descobriremos x. Muito mais fácil.
Sabemos que a velocidade inicial é 1, logo, quanto t = 0, v = 1. Então:
$$v(0) = \dfrac{2x e^0 - 2}{x e^0 + 1} = 1 \\ \dfrac{2x - 2}{x + 1} = 1 \\ 2x - 2 = x + 1 \\ 2x - x = 1 + 2 \\ x = 3$$
Sendo assim, as equações finais ficam, a seu critério, ou:
$$\dfrac{2 + v}{2 - v} = 3e^{32t}$$
Ou:
$$v = \dfrac{6e^{32t} - 2}{3e^{32t} + 1} = 2 \dfrac{(3e^{32t} - 1)}{(3e^{32t} + 1)}$$
E isso é tudo que o Bronson mais antigo tem sobre queda livre. Recomendo mesmo estudarem o primeiro se for cair novamente, os outros dois são muito mais complexos e duvido bastante que ele vá soltar na prova... Mas deixei aqui porque, se vocês entenderam os três, vocês dominaram a matéria e não passarão necessidade na prova de forma alguma.

Bons estudos, galera. Qualquer coisa é só me dar um toque.